PT 19 S4 Q10 Forum

Prepare for the LSAT or discuss it with others in this forum.
Post Reply
chrijani

New
Posts: 20
Joined: Fri Apr 25, 2014 2:36 pm

PT 19 S4 Q10

Post by chrijani » Thu Jun 19, 2014 3:07 pm

I am confused as to why A is the answer choice and not B.
According to Kaplan, they say if A is correct then it'll be a rare candidate who takes a chance on the authors strategy, as the media may only air the opposition and disregard the later support. They further that by stating that media is a huge part of national campaigning. But wouldn't that be an outside assumption we would make. And aren't we only suppose to take what is given within the stimulus to support our answers? Because that is why I disregarded A as an answer, thinking who cares what the media covers because it doesn't limit the SPEECH, the speech is still given and still heard from those attending and such. Where as if you choose B if many people do not find politicians in the first place then that would make practically any strategy they choose ineffective, no one trust them... Can someone help me where my thinking has gone wrong here?

User avatar
Clyde Frog

Platinum
Posts: 8985
Joined: Sun May 26, 2013 2:27 am

Re: PT 19 S4 Q10

Post by Clyde Frog » Thu Jun 19, 2014 4:13 pm

chrijani wrote:I am confused as to why A is the answer choice and not B.
According to Kaplan, they say if A is correct then it'll be a rare candidate who takes a chance on the authors strategy, as the media may only air the opposition and disregard the later support. They further that by stating that media is a huge part of national campaigning. But wouldn't that be an outside assumption we would make. And aren't we only suppose to take what is given within the stimulus to support our answers? Because that is why I disregarded A as an answer, thinking who cares what the media covers because it doesn't limit the SPEECH, the speech is still given and still heard from those attending and such. Where as if you choose B if many people do not find politicians in the first place then that would make practically any strategy they choose ineffective, no one trust them... Can someone help me where my thinking has gone wrong here?
That's weird that they add in something like that.


Anyways, here's my take on the answer choices.

You went wrong with choices (B) since you did not take the information presented to you as it is. It states in the first sentence that audiences find a speech more convincing if they argue for/against their positions. The key word here is audiences. We don't care if many people don't find them convincing or not since we know already that audiences do. In LSAT terms, many could mean two people out of the entire world's population. (A) weakens the recommendation by casting doubt on the effectiveness of the strategy. Lets say that the media only takes the parts of the candidates arguments that they use to argue against their own position, then voters may be a little reluctant to vote for them, which would limit the recommendation's effectiveness.

Not a big fan of Kaplan either. I'd suggest buying the Manhattan guides.

User avatar
WaltGrace83

Silver
Posts: 719
Joined: Fri Apr 12, 2013 5:55 pm

Re: PT 19 S4 Q10

Post by WaltGrace83 » Thu Jun 19, 2014 4:21 pm

chrijani wrote:I am confused as to why A is the answer choice and not B.
According to Kaplan, they say if A is correct then it'll be a rare candidate who takes a chance on the authors strategy, as the media may only air the opposition and disregard the later support. They further that by stating that media is a huge part of national campaigning. But wouldn't that be an outside assumption we would make. And aren't we only suppose to take what is given within the stimulus to support our answers? Because that is why I disregarded A as an answer, thinking who cares what the media covers because it doesn't limit the SPEECH, the speech is still given and still heard from those attending and such. Where as if you choose B if many people do not find politicians in the first place then that would make practically any strategy they choose ineffective, no one trust them... Can someone help me where my thinking has gone wrong here?

I somewhat agree with the bolded. I don't know if we can assume that the media makes a HUGE impact on the national campaigning for strictly LSAT purposes.

However, (B) is actually a strengthener I believe. IF the arguments are one-sided THEN the arguments are not convincing. (B) gives us the negated sufficient condition. While we cannot really assume that negated necessary condition (saying that the arguments are convincing), it seems to make the idea that arguments that are NOT one-sided are convincing a little bit more plausible. We see this all the time in strengthening answer choices. The stimulus gives us A → B and the correct answer choice gives us ~A → ~B. But now I am getting besides the point.

Even if you disagree with that reasoning, you can eliminate (B) for being otherwise irrelevant.

(C) This basically says that ALL speech techniques would not do that much. Thus, while this seems to weaken the idea of giving a speech, it doesn't seem to weaken the specific speech technique.

(D) is a strengthener

(E) irrelevent

So we are left with (A). This isn't a slam dunk because, as you say, we still have to establish that the media means something. However, it is clearly better because it OPENS UP THE POSSIBILITY that the plan could backfire (a likely indicator of a weakener).
  • To me, (C) is actually the most tempting answer.
What do you think?

EDIT: Clyde's response on (B) is better than mine because it is more simple. If you don't understand what I was saying (I am a notoriously bad communicator online sometimes) just forget about it and read the far superior explanation.

Daily_Double

Silver
Posts: 1031
Joined: Tue Dec 04, 2012 8:45 pm

Re: PT 19 S4 Q10

Post by Daily_Double » Thu Jun 19, 2014 4:32 pm

Edit: scooped. Big time.

So here we are trying to limit the effectiveness of something---hey that means we're weakening this recommendation. Awesome, weaken questions are in the Assumption family, and thus the conclusion is not necessarily true. We have to first identify why the conclusion is flawed, then make it more likely that the recommendation shouldn't be adopted. Let's look at the argument:

(1) Presenting reasons against a position taken by an author makes the author appear unbiased
(2) Being unbiased leads audiences to find an author's position more appealing.
______________________________________________________________________
Thus, Politicians who want to win by votes should present reasons against a position they take

Alright, I've switched up the order of the premises because the second sentence supports the first and I like to order things to reflect the structure. I've also changed the words around slightly because pronouns tend to distract people from the relevant phrases of premises.

Now let's talk about what we can infer, then look towards why that inference doesn't establish the conclusion.

What We Can Infer
We know that when an author weakens his own view at the beginning of a speech the author appears impartial and that leads the audience members to find the author more convincing. Thus we can infer that when an author weakens his own view audience members or the people who hear the speech find him more convincing.

Why the Conclusion is Not Necessarily True
Just because the audience members find an author more convincing that they would have thought if he didn't take a certain action doesn't mean that taking that action will lead people to vote for him. So the error is that the author of this argument assumes:

"There is some relationship between the persuasion level of the listeners of an author's speech and the likelihood of that author winning an election such that when the persuasion level increases (people find the author more convincing), the author is more likely to get enough votes to be successful."

Because if this is not true, if there is no relationship between how convinced the audience of a speech is to the likelihood of that author winning an election, then it's not absolutely true that the author should weaken his view at the beginning of a presentation.

Alright, now we've spotted the flaw, let's prephrase something that would weaken the conclusion. Well the effect that we want is a statement that shows how weakening a position or the level the audience/listeners of a speech are persuaded doesn't lead towards the author winning an election or gaining votes. That's a pretty vague prephrase, but that's generally what you want to be thinking in Weaken and Strengthen questions---since they're a bit less predictable, you should prephrase the effect of the correct answers instead of thinking about the exact way the answer arrives at that effect.

Answer choice (A) points out that taking this action (weakening a position at the beginning of a presentation) doesn't always lead to the desired effect (people hearing/watching the speech are more convinced) because the news media might not even show that part. Because there's no guarantee the first part of an author's presentation, which is the part where this action takes place, will be shown, there's no guarantee the desired effect will take place. This weakens by showing that people might not be convinced of an author's position because they might not hear the author weaken his own view at the beginning of the presentation.

As a side note, I really don't care too much for Kaplan's explanation you provided. The reason it weakens has more to do with the fact that under this answer, we don't know if the premises even apply to the situation. It touches on the major flaw in a way by showing that the desired effect, which the author assumes leads to the truth of the conclusion, may not take place.

Answer choice (B) doesn't address the success of the recommendation in the conclusion and for that reason, it doesn't weaken it. Under this answer, all politicians' positions are in doubt from the beginning because, for one reason, people think politicians are one-sided. If anything this might strengthen (to a very small degree) the argument to weaken a position at the beginning of a presentation because doing so would make the author appear unbiased, which might make votes see him as not one-sided.

Clear things up?

User avatar
Clyde Frog

Platinum
Posts: 8985
Joined: Sun May 26, 2013 2:27 am

Re: PT 19 S4 Q10

Post by Clyde Frog » Thu Jun 19, 2014 4:48 pm

It annoys me how the early lsat question stems are worded, as if to be overly complicated. I like the standard "which one of the following, if true, most seriously weakens the argument?" question stem.

Want to continue reading?

Register now to search topics and post comments!

Absolutely FREE!


User avatar
Jeffort

Gold
Posts: 1888
Joined: Wed Jun 18, 2008 4:43 pm

Re: PT 19 S4 Q10

Post by Jeffort » Thu Jun 19, 2014 5:44 pm

The media being involved and playing a big role in national elections is a warranted/allowed assumption since it is a common sense fact about reality that every adult college educated person knows is a fact that is always true in reality.

The technique for convincing people to agree with you/your positions described in the support requires the 'audience' to hear the entire speech for it to be effective. According to the evidence, the technique IS very effective for persuading the audience 'makes the speaker appear fair-minded and trustworthy' and the cause is stated: since they started the speech by giving arguments against their position before giving reasons to accept it. If a person doesn't hear/listen to/read the entire speech to hear both the pro and con arguments for the candidates position, the cause that would make the speech convincing to the audience is absent, thus limiting the effectiveness of the recommended strategy for running for national political office where you have to reach an audience size that's waaaaaayyyyyy more people than you could ever humanly speak to in person to get lots o votes.

It's key to realize that running for national office means that the voters are spread over a large geographic area and that there are a LOT of potential voters (in the USA or Canada that means millions of people) since national=nation=country. Common sense dictates that it's impossible for one human to be able to give in person speeches to audiences of millions of people or anywhere close to a majority of the population of the nation/people that are eligible voters where you have them in person in the audience sitting there to listen to your whole speech. Therefore, to be part of the 'audience' to a speech(es) made by the candidate and be influenced by it, citizens have to depend on hearsay or media news sources to hear or get any news/information about speeches the candidate gives unless they were physically present at the event when the aspiring politician spoke and listened to the whole speech.

(A) points out that candidates don't have control over what the media reports and how they decide to edit/select which parts of the speech to broadcast/report about. It pretty much just describes the norm of what media/news sources frequently do, selectively edit and broadcast 'sound bytes'/clips/parts of speeches rather than playing the whole thing. Turn on any news channel any time/day of the week and you'll see examples of this common media practice.

If the media doesn't broadcast/report/print the entire speech, then most voters will not hear/read it with both the pros and cons parts and won't be exposed to the cause that makes the technique effective for persuading people.

(B) actually gives you a good reason to support why the technique as described IS very effective, because by addressing arguments against his/her position, the candidate isn't being one-sided.

chrijani

New
Posts: 20
Joined: Fri Apr 25, 2014 2:36 pm

Re: PT 19 S4 Q10

Post by chrijani » Fri Jun 20, 2014 9:20 am

Wow! Thank you so much! All of you have provided amazing responses. I understand the question now and can see a few things I overlooked that lead me to the wrong answer. Such as, "national" "audience" (with media limiting one potential audience). I guess that is one of the major issues I have is overlooking things or not fully understanding what is being presented in the stimulus. Anyways, I appreciate all the responses and thank you for taking the time to help me.

Want to continue reading?

Register for access!

Did I mention it was FREE ?


Post Reply

Return to “LSAT Prep and Discussion Forum”